Side 1 av 1

Likning og sirkel på polar form

Lagt inn: 04/04-2012 15:48
av Nebuchadnezzar
Jeg har en matlab øving nå, og skal skrive om et området R som er over linjen

[tex]y = \frac{x}{3} + 2[/tex] og under buen [tex]y = 3 + \sqrt{9-x^2} [/tex] til polar form.

Er det så enkelt at jeg bare benytter meg av [tex]\cos(\theta)[/tex] og [tex]\sin(\theta)[/tex] ?

Slik at jeg ender opp med [tex]r = \left( \frac13 \cos \theta + 2\right)^2 + \cos^2\theta [/tex] for linjen, og

[tex]r = \left( 3 + 3 \cos^2 \theta\right)^2 + 3^2 \cos^2 \theta[/tex] for sirkelen?

Jeg tegnet begge disse, men det så litt rart ut.

Om dette er riktig, blir integralet mitt da

[tex]\int_R f(x,y) \, \mathrm{d}x = \int_{0}^{\pi} \ \ \int_{\frac{\cos \theta}{3}+2}^{\left( \frac13 \cos \theta + 2\right)^2 + \cos^2 \theta} f(r,\theta) r \, \mathrm{dr} \, \mathrm{d}\theta[/tex]

?(ser litt rart ut) Takk på forhånd, for all hjelp =)

Lagt inn: 04/04-2012 16:02
av Aleks855
Det var de merkeligste grensene jeg har sett på et integral :lol:

Lagt inn: 04/04-2012 16:06
av Nebuchadnezzar
Fant endelig ut av ting, nå tror jeg det skal bli riktig...

[tex]\int_{0}^{\pi} \int_{-6/(-3\sin(\theta)+\cos(\theta))}^{6\sin \theta} f(r,\theta) r \, \mathrm{d}r \, \mathrm{d}\theta[/tex]

Skulle noen ganger ønske jeg kunne slette tåpelige innlegg, men sånn er livet =)

EDIT: Dette gikk i dass...

Prøvde å regne ut integralet først med kartesiske koordinter og så med polar.

Oppgaven bad om å integrere

http://www.math.ntnu.no/emner/MA1103/20 ... atlab2.pdf

[tex]f(x,y) = \frac{x}{\sqrt{x^2 + y^2}}[/tex] over [tex]R[/tex], hvor

[tex]R[/tex] er området under kurven [tex]y = 3 + \sqrt{9-x^2}[/tex] der [tex]|x|<3[/tex] og over linjen [tex]y = x/3 + 2 [/tex]

I kartesiske tenkte jeg at integralet ble som følger

[tex]A = \int_{-3}^{3} \int_{x/3 + 2}^{3 + \sqrt{9-x^2}} f(x,y) \approx -2.01[/tex]

Ble dette rett?

Så spurte oppgaven om å regne ut det samme dobbeltintegralet men nå ved hjelp av polarkoordinater. Da tenkte jeg at likningen for kurven ble

[tex]y = 3 + \sqrt{9 - x^2} \ , \ |x| \quad \Leftrightarrow \quad r_2 = 6 \sin \theta \ , \, \pi/4 < \theta < 3\pi/4[/tex]

Mens likningen for linjen blir

[tex]y = x/3 + 2 \quad \Leftrightarrow \quad r_1 = \frac{6}{3 \sin \theta - \cos \theta}[/tex]

Slik at dobbeltintegralet blir

[tex]\int_{\pi/4}^{3\pi/4} \int_{r_1}^{r_2} f(x,y) r \, \mathrm{d}r \, \mathrm{d}\theta \, \approx \, 17.28 [/tex]

Men som den eventuellt oppmerksomme leser legger merke til så er ikke svarene like. De burde dog være like. Men jeg tror rett og slett grenesene mine er feile. Dog vet jeg ikke hvordan jeg fikser dette.

Vet noen hvordan jeg kan regne ut dette dobbeltintegralet på riktig måte ved bruk av polarkoordinater?

Lagt inn: 06/04-2012 14:47
av drgz
Det står i oppgaveteksten at området ditt er gitt av
[tex]\mathcal{D}=\left\{(x,y)|0\leq x\leq 3,x/3+2\leq y\leq 3+\sqrt{9-x^2}\right\}[/tex], så svaret du har kommet fram til kan ikke stemme ettersom du har feil grenser på det ytterste integralet. Det kan også tenkes at det enkleste her er å bytte om rekkefølgen, å gå fra dydx til dxdy; f(x,y) integrert mhp x blir ganske fint; integrert mhp y blir det verre.

Numerisk i Matlab derimot:

Kode: Velg alt

f=@(x,y) x./sqrt(x.^2+y.^2);
ymin=@(x) x/3+2;
ymax=@(x) 3+sqrt(9-x.^2);
Q = quad2d(f,0,3,ymin,ymax)
gir

Q = 2.4956

Hvis du tegner området i (x,y)-planet, og deretter holder [tex]\theta[/tex] konstant mens du tegner en vektor [tex]\vec{r}[/tex] fra origo som går gjennom området, så vil du se at din nedre grense for [tex]r[/tex] vil være linjen [tex]y=x/3+2 = 2/(\sin(\theta)-\cos(\theta)/3)[/tex], og at den øvre grensen til være sirkelen [tex]x^2+(y-3)^2=3^2=2\cdot3\sin(\theta)=6\sin(\theta)[/tex].

Lar du deretter [tex]r[/tex] være konstant og tegner en vektor [tex]\vec{\theta}[/tex] som går fra 0 og langs en sirkel mot klokken, så vil du se at den nedre grensen for [tex]\theta[/tex] vil være [tex]\pi/4[/tex] (fra punktet x=3,y=3), mens den øvre blir [tex]\pi/2[/tex].

Dermed får du [tex]f(x,y)\,\mathrm{d}A=r\cos(\theta)\,\mathrm{d}r\,\mathrm{d}\theta[/tex], og
[tex]\mathcal{D}=\left\{(r,\theta)|2/(\sin(\theta)-\cos(\theta)/3)\leq r\leq 6\sin(\theta),\pi/4\leq\theta\leq \pi/2\right\}[/tex]

Integrerer du dette skal du få samme svar som med de kartesiske koordinatene (du kan sjekke her: http://www.wolframalpha.com/widgets/vie ... 2fad9ebddc).

Lagt inn: 06/04-2012 23:12
av Nebuchadnezzar
Skal se på det du skrev, flaut at jeg ikke så at det var til høyre for y-aksen.

Tusen takk for svar!

Om jeg skal snu grensene, hvordan blir det da? Syntes fortsatt dette med grensene er vanskelig. Ser åpenbart at [tex]2<y<6[/tex], men hva x går mellom blir vanskelig. Må jeg dele den opp i to deler?

Blir det kanskje

[tex]\int_2^3 \int_0^{3y-6} f(x,y)\,dx\,dy+ \int_{3}^{6} \int_{\sqrt{6y-y^2}}^{0} f(x,y) \, dx \, dy[/tex] ?

Lagt inn: 07/04-2012 00:20
av drgz
Jeg ser nå at du ikke blir bedt om å regne ut noe eksplisitt svar for hånd, så da går det jo an å la det være. Jeg bare sjekket fort på innerste ingegralet og så fort at hvis man integrerte f(x,y) mhp y, så ville det bli noe mer vanskelig å regne videre etter å ha satt inn grensene; mens ved å integrere mhp x så ble det mye finere uttrykk.

Er noen år siden jeg har holdt på å snu grenser, men grensene du har satt opp ser fornuftige ut (uten at jeg får sjekket med det jeg gjorde hjemme tidligere i dag).